logo

ルベーグ空間におけるコーシー・シュワルツの不等式 📂ルベーグ空間

ルベーグ空間におけるコーシー・シュワルツの不等式

概要1

$f,g \in L^{2} (E)$ の場合 $fg \in L^{1}(E)$ が成り立ち、以下が当てはまる。

$$ \left| \int_{E} f \overline{g} dm \right| \le \left\| f g \right\|_{1} \le \left\| f \right\|_{2} \left\| g \right\|_{2} $$

ここで、$\| \cdot \|_{2}$は$L^{2}$空間のノルム、$\| \cdot \|_{1}$は$L^{1}$空間のノルムだ。

説明

関数解析を学んでいるなら、この不等式がなぜコーシー・シュヴァルツと呼ばれているかすぐに理解できるはずだ。実際、内積が定義されている場所であれば、コーシー・シュヴァルツの不等式をどこでも見つけることができる。ヘルダーの不等式へと一般化することができる。

証明

$$ \int_{E} fg dm \le \int_{E} |fg| dm \le \int_{E} | f + g |^2 dm < \infty $$

従って、$fg \in L^{1}$。一方、$\displaystyle (x - y)^2 \ge 0$から以下のことが得られる。

$$ xy \le \dfrac{1}{2} \left( x^2 + y^2 \right) $$

  • ケース1. $\left\| f \right\|_{2} = 0$ または $\left\| g \right\|_{2} = 0$の場合

    ほぼどこでも$f = 0$か部分的に$g = 0$なので、ほぼどこでも$f\overline{g} = 0$。従って、$\displaystyle \left| \int_{E} f \overline{g} dm \right| = \left\| fg \right\|_{1} = 0$であり、$\left\| f \right\|_{2} \left\| g \right\|_{2} = 0$であるから、不等式を満たす。

  • ケース2. $\left\| f \right\|_{2} = \left\| g \right\|_{2} = 1$の場合

    $$ \left| \int_{E} f \overline{g} dm \right| \le \int_{E} \left| f \overline{g} \right| dm = \left\| fg \right\|_{1} \le {{1} \over {2}} (1 + 1) = 1 = \left\| f \right\|_{2} \left\| g \right\|_{2} $$

    従って、不等式を満たす。

  • ケース3. その他のケース

    正規化された関数$\displaystyle \hat{ f } : = {{f} \over {\left\| f \right\|_{2}}}$と$\displaystyle \hat{ g } : = {{g} \over {\left\| g \right\|_{2}}}$を新たに定義しよう。そうするとケース2より

    $$ \left| \int_{E} \hat{f} \overline{\hat{g} } dm \right| \le \left\| \hat{f} \hat{g} \right\|_{1} \le \left\| \hat{f} \right\|_{2} \left\| \hat{g} \right\|_{2} $$

    展開すると

    $$ \left| \int_{E} {{f} \over {\left\| f \right\|_{2}}} \overline{{{g} \over {\left\| g \right\|_{2}}} } dm \right| \le \left\| {{f} \over { \left\| f \right\|_{2}}} {{g} \over {\left\| g \right\|_{2}}} \right\|_{1} \le \left\| {{f} \over {\left\| f \right\|_{2}}} \right\|_{2} \left\| {{g} \over {\left\| g \right\|_{2}}} \right\|_{2} $$

    スカラー$\left\| f \right\|_{2} , \left\| g \right\|_{2} \in (0, \infty)$を整理すると

    $$ \left| \int_{E} f \overline{g} dm \right| \le \left\| f g \right\|_{1} \le \left\| f \right\|_{2} \left\| g \right\|_{2} $$

    を得る。

関連項目


  1. Capinski. (1999). Measure, Integral and Probability: p132. ↩︎